-
Notifications
You must be signed in to change notification settings - Fork 2
/
lista06.tex
316 lines (293 loc) · 20 KB
/
lista06.tex
1
2
3
4
5
6
7
8
9
10
11
12
13
14
15
16
17
18
19
20
21
22
23
24
25
26
27
28
29
30
31
32
33
34
35
36
37
38
39
40
41
42
43
44
45
46
47
48
49
50
51
52
53
54
55
56
57
58
59
60
61
62
63
64
65
66
67
68
69
70
71
72
73
74
75
76
77
78
79
80
81
82
83
84
85
86
87
88
89
90
91
92
93
94
95
96
97
98
99
100
101
102
103
104
105
106
107
108
109
110
111
112
113
114
115
116
117
118
119
120
121
122
123
124
125
126
127
128
129
130
131
132
133
134
135
136
137
138
139
140
141
142
143
144
145
146
147
148
149
150
151
152
153
154
155
156
157
158
159
160
161
162
163
164
165
166
167
168
169
170
171
172
173
174
175
176
177
178
179
180
181
182
183
184
185
186
187
188
189
190
191
192
193
194
195
196
197
198
199
200
201
202
203
204
205
206
207
208
209
210
211
212
213
214
215
216
217
218
219
220
221
222
223
224
225
226
227
228
229
230
231
232
233
234
235
236
237
238
239
240
241
242
243
244
245
246
247
248
249
250
251
252
253
254
255
256
257
258
259
260
261
262
263
264
265
266
267
268
269
270
271
272
273
274
275
276
277
278
279
280
281
282
283
284
285
286
287
288
289
290
291
292
293
294
295
296
297
298
299
300
301
302
303
304
305
306
307
308
309
310
311
312
313
314
315
316
% Filename: lista06.tex
%
% This code is part of 'Solutions for MT402, Matrizes'
%
% Description: This file corresponds to the solutions of homework sheet 06.
%
% Created: 25.03.12 04:31:30 PM
% Last Change: 04.06.12 05:11:54 PM
%
% Authors:
% - Raniere Silva (2012): initial version
%
% Copyright (c) 2012 Raniere Silva <r.gaia.cs@gmail.com>
%
% This work is licensed under the Creative Commons Attribution-ShareAlike 3.0 Unported License. To view a copy of this license, visit http://creativecommons.org/licenses/by-sa/3.0/ or send a letter to Creative Commons, 444 Castro Street, Suite 900, Mountain View, California, 94041, USA.
%
% This work is distributed in the hope that it will be useful, but WITHOUT ANY WARRANTY; without even the implied warranty of MERCHANTABILITY or FITNESS FOR A PARTICULAR PURPOSE.
%
\begin{questions}
\question A elimina\c{c}\~{a}o dos elementso $A_{ik}, i = k + 1, \ldots, n$ \'{e} realizada atrav\'{e}s de uma sequ\^{e}ncia de opera\c{c}\~{o}es elementares: multiplicar a linha $k$ por $m_{ik}$ e subtrair o resultado da linha $i$, $i = k + 1, \ldots, n$. Esta sequ\^{e}ncia de opera\c{c}\~{o}es comp\~{o}e a Transforma\c{c}\~{a}o de Gauss, $M_k$ da etapa $k$. Demonstre que:
\begin{parts}
\part Cada opera\c{c}\~{a}o elementar corresponde a pr\'{e}-multiplicar a matriz por $E_i = I - m_{ik} e_i e_k^t$;
\begin{solution}
Temos que
\begin{align*}
E_i A &= \left( I - m_{ik} e_i e_k^t \right) A \\
&= A - m_{ik} e_i e_k^t A \\
&= A - e_i \left( m_{ik} A_{k \mdot}^t \right).
\end{align*}
\end{solution}
\part $M_k = E_n E_{n - 1} \ldots E_{k + 1} = I - \sum_{i = k + 1}^n m_{ik} e_i e_k^t = I - \gamma_k e_k^t$, onde $\gamma_k : n \times 1$, $\gamma_k(i) = 0, i = 1, \ldots, k$ e $\gamma_k(i) = m_{ik}, i = k + 1, \ldots, n$.
\begin{solution}
Temos que
\begin{align*}
M_k &= E_n E_{n - 1} \ldots E_{k + 1} \\
&= \left( I - m_{n, k} e_n e_k^t \right) \left( I - m_{n - 1, k} e_{n-1} e_k^t \right) \ldots \left( I - m_{k + 1, k} e_{k + 1} e_k^t \right) \\
% &= \left( I - m_{n - 1, k} e_{n - 1} e_k^t - m_{n, k} e_n e_k^t + m_{n, k} m_{n - 1, k} e_n e_k^t e_{n - 1} e_k^t \right) \ldots \left( I - m_{k + 1, k} e_{k + 1} e_k^t \right) \\
&= \left( I - m_{n - 1, k} e_{n - 1} e_k^t - m_{n, k} e_n e_k^t \right) \ldots \left( I - m_{k + 1, k} e_{k + 1} e_k^t \right) && e_n e_k^t = 0 \\
&= I - \sum_{i = k + 1}^n m_{ik} e_i e_k^t \\
&= I - \left( \sum_{i = k + 1}^n m_{ik} e_i \right) e_k^t \\
&= I - \gamma_k e_k^t,
\end{align*}
onde $\gamma_k : n \times 1$, $\gamma_k(i) = 0, i = 1, \ldots, k$ e $\gamma_k(i) = m_{ik}, i = k + 1, \ldots, n$.
\end{solution}
\end{parts}
\question Demonstre que a matriz $L = \left( M_{n - 1} M_{n - 2} \ldots M_2 M_1 \right)^{-1} = I + \sum_{k = 1}^n \gamma_k e_k^t$, se os fatores $L$ e $U$ s\~{a}o calculados pelo processo de Elimina\c{c}\~{a}o de Gauss sem estrat\'{e}gia de pivoteamento parcial.
\begin{solution}
Temos que
\begin{align*}
L &= \left( M_{n - 1} M_{n - 2} \ldots M_2 M_1 \right)^{-1} \\
&= M_1^{-1} M_2^{-1} \ldots M_{n - 2}^{-1} M_{n - 1}^{-1}
\end{align*}
Mas pelo exerc\'{i}cio 1(b) temos que $M_k = I - \gamma_k e_k^t$ de onde verificamos que
\begin{align*}
M_k M_k^{-1} &= \left( I - \gamma_k e_k^t \right) \left( I + \gamma_k e_k^t \right) \\
&= I + \gamma+k e_k^t - \gamma_k e_k^t - \gamma_k e_k^t \gamma_k e_k^t \\
&= I - \gamma_k e_k^t \gamma_k e_k^t \\
&= I && e_k^t \gamma_k = 0.
\end{align*}
Logo,
\begin{align*}
L&= \left( I + \gamma_1 e_1^t \right) \left( I + \gamma_2 e_2^t \right) \ldots \left( I + \gamma_{n - 2} e_{n - 2}^t \right) \left( I + \gamma_{n - 1} e_{n - 1}^t \right) \\
&= \left( I + \gamma_1 e_1^t + \gamma_2 e_2^t + \gamma_1 e_1^t \gamma_2 e_2^t \right) \ldots \left( I + \gamma_{n - 2} e_{n - 2}^t \right) \left( I + \gamma_{n - 1} e_{n - 1}^t \right) \\
&= \left( I + \gamma_1 e_1^t + \gamma_2 e_2^t \right) \ldots \left( I + \gamma_{n - 2} e_{n - 2}^t \right) \left( I + \gamma_{n - 1} e_{n - 1}^t \right) \\
&= I + \sum_{k = 1}^{n - 1} \gamma_k e_k^t.
\end{align*}
\end{solution}
\question $P = I - u u^t$, $u = e_r - e_s$ representa a permuta\c{c}\~{a}o das linhas $r$ e $s$:
\begin{parts}
\part Demonstre que $P^{-1} = P$ e portanto, $P^2 = I$;
\begin{solution}
Se $P$ corresponde a permuta\c{c}\~{a}o das linhas $r$ e $s$ ent\~{a}o $P^{-1}$, a opera\c{c}\~{a}o inversa, corresponde a permuta\c{c}\~{a}o das linhas $s$ e $r$ que equivale a permuta\c{c}\~{a}o das linhas $r$ e $s$, logo $P^{-1} = P$.
Portanto,
\begin{align*}
P P^{-1} &= P P \\
&= \left( I - u u^t \right) \left( I - u u^t \right) \\
&= I - u u^t - u u^t + u u^t u u^t \\
&= I - 2 u u^t + \| u \|_2^2 u u^t \\
&= I - 2 u u^t + 2 u u^t && u = e_r - e_s \\
&= I.
\end{align*}
\end{solution}
\part Demonstre que $P M_k P = I - \tilde{\gamma}_k e_k^t = M_k$ onde $\tilde{\gamma}_k$ \'{e} o vetor $\gamma_k$ com os multiplicadores $m_{rk}$ e $m_{sk}$, $r > k$ e $s > k$, permutados.
\begin{solution}
Pelo exerc\'{i}cio 1(b) temos que $M_k = I - \gamma_k e_k^t$ e portanto
\begin{align*}
P M_k P &= \left( I - u u^t \right) \left( I - \gamma_k e_k^t \right) \left( I - u u^t \right) \\
&= \left( I - \gamma_k e_k^t - u u^t + u u^t \gamma_k e_k^t \right) \left( I - u u^t \right) \\
&= \left( I - \gamma_k e_k^t - u u^t + \left( \gamma_k(r) - \gamma(s) \right) u e_k^t \right) \left( I - u u^t \right) \\
\begin{split}
&= I - u u^t - \gamma_k e_k^t - \gamma_k e_k^t u u^t - u u^t + u u^t u u^t \\ &\quad {}+ \left( \gamma_k(r) - \gamma_k(s) \right) u e_k^t - \left( \gamma_k(r) - \gamma_k(s) \right) u e_k^t u u^t
\end{split} \\
\begin{split}
&= I - u u^t - \gamma_k e_k^t - u u^t + 2 u u^t + \left( \gamma_k(r) - \gamma_k(s) \right) u e_k^t
\end{split} && e_k^t u = 0, r > k, s > k \\
&= I - \gamma_k e_k^t + \left( \gamma_k(r) - \gamma_k(s) \right) u e_k^t \\
&= I - \left( \gamma_k - \left( \gamma_k(r) - \gamma_k(s) \right) u \right) e_k^t \\
&= I - \tilde{\gamma}_k e_k^t,
\end{align*}
onde $\tilde{\gamma}_k$ corresponde ao vetor $\gamma_k$ com os multiplicadores $m_{rk}$ e $m_{sk}$ permutados.
\end{solution}
\end{parts}
\question[Ver Teorema 3.4.1, p\'{a}gina 113, do Golub\nocite{Golub:1996:matrix}] Se os fatores $L$ e $U$ s\~{a}o obtidos atrav\'{e}s da Elimina\c{c}\~{a}o de Gauss com pivoteamento parcial, represente $P$ e $L$ de acordo com as permuta\c{c}\~{o}es e transforma\c{c}\~{o}es de Gauss realizadas durante o processo.
\begin{solution}
Pelo exerc\'{i}cio 2 temos que, no caso em que n\~{a}o \'{e} utilizado permuta\c{c}\~{a}o,
\begin{align*}
L &= \left( M_{n - 1} M_{n - 2} \ldots M_2 M_1 \right)^{-1}.
\end{align*}
J\'{a} ao utilizarmos uma permuta\c{c}\~{a}o antes de cada opera\c{c}\~{a}o elementar temos
\begin{align*}
L &= \left( M_{n - 1} P_{n - 1} M_{n - 2} P_{n - 2} \ldots M_2 P_2 M_1 P_1 \right)^{-1},
\end{align*}
onde $P_k$ \'{e} a permuta\c{c}\~{a}o efetuada antes de realizar a opera\c{c}\~{a}o elementar $M_k$.
\end{solution}
\question Se $P A = L U$, como usar estes fatores para resolver o sistema linear $A^t x = b$?
\begin{solution}
Se $P A = L U$ ent\~{a}o $A = P^t L U$. Logo,
\begin{align*}
A^t x &= \left( P^t L U \right) x \\
&= U^t L^t P x.
\end{align*}
E para resolver o sistema utilizamos $x = P^t L^{-t} U^{-t} b$.
\end{solution}
\question Descreva como o processo de elimina\c{c}\~{a}o de Gauss com pivoteamente parcial pode ser usando para obter o determinante de uma matriz $A : n \times n$.
\begin{solution}
Seja $P A = L U$, ent\~{a}o
\begin{align*}
\det(P A) &= \det(L U) \\
&= \det(L) \det (U) \\
&= \prod_{i = 1}^n l_{ii} \prod_{i = 1}^n u_{ii} && \text{pois L e U s\~{a}o diagonais} \\
&= \prod_{i = 1}^n u_{ii} && \text{pois L possui diagonal unit\'{a}ria}.
\end{align*}
Mas
\begin{align*}
\det(P A) &= \det(P) \det(A) \\
&= (-1)^{c} \det(A),
\end{align*}
onde $c$ \'{e} o n\'{u}mero de permuta\c{c}\~{o}es realizadas. Logo, $\det(A) = (-1)^c \prod_{i = 1}^n u_{ii}$.
\end{solution}
\question(Ver equa\c{c}\~{a}o 3.10.12, p\'{a}gina 149, do Meyer\nocite{Meyer:2000:matrix}] Sobre o teorema da exist\^{e}ncia e unicidade da fatora\c{c}\~{a}o LU: se uma submatriz principal dominante de ordem $k$ for singular, podemos afirmar que a matriz $A$ n\~{a}o tem fatora\c{c}\~{a}o $LU$? Fundamente sua resposta teoricamente e com exemplos.
\begin{solution}
O Teorema 3.2.1, p\'{a}gina 97, do Golub\nocite{Golub:1996:matrix} trata da exist\^{e}ncia e unicidade da fatora\c{c}\~{a}o LU e \'{e} transcrito abaixo:
\begin{quote}
$A \in \mathbb{R}{^n \times n}$ possue fatora\c{c}\~{a}o LU se $\det(A(1:k, 1:k)) \neq 0$ para $k = 1:n - 1$. Se a fatora\c{c}\~{a}o LU existe e $A$ \'{e} n\~{a}o singular, ent\~{a}o a fatora\c{c}\~{a}o LU \'{e} \'{u}nica e $\det(A) = \prod_{i = 1}^n u_{[ii}$.
\end{quote}
Como podemos notar, o teorema apenas garante que se todas as submatrizes principais s\~{a}o n\~{a}o singulares ent\~{a}o a fatora\c{c}\~{a}o LU existe e nada \'{e} afirmada para o caso de alguma das submatrizes principais ser singular.
Pelo contra-exemplo abaixo,
\begin{align*}
A = \begin{bmatrix}
1 & 1 \\
1 & 0
\end{bmatrix} = \begin{bmatrix}
1 & 0 \\
1 & 1
\end{bmatrix} \begin{bmatrix}
1 & 1 \\
0 & -1
\end{bmatrix} = L U,
\end{align*}
mostramos que mesmo existindo uma submatriz singular a fatora\c{c}\~{a}o LU ainda existe.
\end{solution}
\question Descreva como obter a inversa de uma matriz $A$ atrav\'{e}s da resolu\c{c}\~{a}o de $n$ sistemas lineares. A fatora\c{c}\~{a}o $LU$ com pivoteamento parcial \'{e} indicada para esta resolu\c{c}\~{a}o? Qual o número total de opera\c{c}\~{o}es necess\'{a}rias para obter $A^{-1}$?
\begin{solution}
Seja $B = A^{-1}$. Podemos obter a matriz $B$ resolvendo os seguintes sistemas lineares:
\begin{align*}
A B_{\mdot i} = e_i, \forall i \in \left\{ 1, 2, \ldots, n \right\}.
\end{align*}
A fatora\c{c}\~{a}o LU com pivoteamento parcial \'{e} indicada para esta resolu\c{c}\~{a}o pois temos que resolver $n$ sistemas lineares que diferem apenas no lado direito e ao utilizarmos pivoteamento parcial temos garantia de encontrar a fatora\c{c}\~{a}o LU da matriz $A$.
Para obter a fatora\c{c}\~{a}o LU necessitamos de $2 n^3 / 3$ opera\c{c}\~{o}es e para resolver um sistema triangular de $n^2$ opera\c{c}\~{o}es. Como para obter a matriz $B$ efetuamos uma fatora\c{c}\~{a}o LU e $2 n$ resolu\c{c}\~{o}es de sistemas triangulares temos que o n\'{u}mero total de opera\c{c}\~{o}es \'{e} $2 n^3 / 3 + 2 n^3$.
\end{solution}
\question Supor que s\~{a}o dados $A : n \times n$, $d: n \times 1$ e $c : n \times 1$, e o objetivo \'{e} calcular: $z = c^t A^{-1} d$. Na express\~{a}o de $z$ a dificuldade est\'{a} no c\'{a}lculo de $s = A^{-1} d$. Analise os dois processos para obter $s$:
\begin{enumerate}
\item invertendo a matriz $A$ e calculando $s = A^{-1} d$ e
\item resolvendo o sistema linear $A s = d$.
\end{enumerate}
Qual das duas formas deve ser escolhida de modo que $z$ seja obtido de modo econômico? Conclus\~{a}o: \'{e} melhor resolver um sistema linear do que inverter uma matriz? Justifique!
\begin{solution}
Deve escolher resolver o sistema linear $A s = d$ pois este processo requer um n\'{u}mero de opera\c{c}\~{o}es pr\'{o}ximo de $2 n^3 / 3$ (realizando uma fatora\c{c}\~{a}o LU e a resolu\c{c}\~{a}o de um sistema linear) enquanto que apenas inverter a matriz $A$ requer um n\'{u}mero de opera\c{c}\~{o}es pr\'{o}ximo de $7 n^3 / 3$ opera\c{c}\~{o}es (realizando uma fatora\c{c}\~{a}o LU e a resolu\c{c}\~{a}o de $n$ sistemas lineares, ver exerc\'{i}cio 8).
\end{solution}
\question Supor que a matriz $A : n \times n$ \'{e} particionada na forma
\[
A = \begin{bmatrix}
A_{11} & A_{12} \\
A_{12} & A_{22}
\end{bmatrix}
\]
onde $A_{11} : r \times s$ \'{e} n\~{a}o singular. Denote por $S$, o complemento de Schr de $A_{11}$ em $A$. Demonstre que $A$ \'{e} n\~{a}o singular, se e somente se $S = A_{22} - A_{21} A_{11}^{-1} A_{12}$ \'{e} n\~{a}o singular. Sugest\~{a}o: escreva a fatora\c{c}\~{a}o $LU$ de $A$ em fun\c{c}\~{a}o das submatrizes $A_{ij}$.
\begin{solution}
Temos que a fatora\c{c}\~{a}o LU da matriz $A$ em termos de sua submatrizes \'{e} dado por
\begin{align*}
A = \begin{bmatrix}
A_{11} & A_{12} \\
A_{21} & A_{22}
\end{bmatrix} = \begin{bmatrix}
I & 0 \\
-A_{21} A_{11}^{-1} & I
\end{bmatrix} \begin{bmatrix}
A_{11} & A_{12} \\
0 & A_{22} - A_{21} A_{11}^{-1} A_{12}
\end{bmatrix},
\end{align*}
onde $A_{22} - A_{21} A_{11}^{-1} A_{12} = S$ \'{e} o complemento de Schur de $A_{11}$.
Para que $A$ seja n\~{a}o singular devemos ter que $\det(A) \neq 0$. Logo,
\begin{align*}
\det(A) &= \det(L U) \\
&= \det(L) \det(U) \\
&= 1 \det(U) && \text{L \'{e} triangular unit\'{a}ria} \\
&= \det(A_{11}) \det(S) && \text{U \'{e} triangular}.
\end{align*}
Portanto, $A$ \'{e} n\~{a}o singular se e somente se $S$ \'{e} n\~{a}o singular.
\end{solution}
\question[Teorema 3.4.3, p\'{a}gina 120, do Golub\nocite{Golub:1996:matrix}] Considere $A : n \times n$, estritamente diagonal dominante por colunas, isto \'{e}: $| a_{ij} | > \sum_{j \neq k} | a_{jk} |$. Demonstre que a fatora\c{c}\~{a}o $LU$ com pivoteamente parcial aplicada sobre a matriz $A$, n\~{a}o ir\'{a} realizar nenhuma permuta\c{c}\~{a}o de linhas, ou seja, ao final do processo a matriz $P$ ser\'{a} a identidade.
\begin{solution}
Particionando a matriz $A$ como
\begin{align*}
A = \begin{bmatrix}
\alpha & w^t \\
v & C
\end{bmatrix},
\end{align*}
onde $\alpha \in \mathbb{R}$. Ap\'{o}s uma itera\c{c}\~{a}o da fatora\c{c}\~{a}o LU temos
\begin{align*}
\begin{bmatrix}
\alpha & w^t \\
v & C
\end{bmatrix} = \begin{bmatrix}
1 & 0 \\
v / \alpha & 1
\end{bmatrix} \begin{bmatrix}
1 & 0 \\
0 & C - v w^t / \alpha
\end{bmatrix} \begin{bmatrix}
\alpha & w^t \\
0 & I
\end{bmatrix}.
\end{align*}
Segue por indi\c{c}\~{a}o em $n$ que se $B = C - v w^t / \alpha$ \'{e} estritamente diagonal dominante ent\~{a}o $A$ possue fatora\c{c}\~{a}o LU, pois se $B = L_1 U_1$ ent\~{a}o
\begin{align*}
A = \begin{bmatrix}
1 & 0 \\
v / \alpha & L_1
\end{bmatrix} \begin{bmatrix}
\alpha & w^t \\
0 & U_1
\end{bmatrix} = L U.
\end{align*}
Para provar que $B$ \'{e} estritamente diagonal dominante, segue da defini\c{c}\~{a}o que
\begin{align*}
\sum_{i = 1, i \neq j}^{n - 1} | b_{ij} | &= \sum_{i = 1, i \neq j}^{n - 1} | c_{ij} - v_i w_j / \alpha | \\
&\leq \sum_{i = 1, i \neq j}^{n - 1} | c_{ij} | + \left( | w_j | / | \alpha | \right) \sum_{i = 1, i \neq j}^{n - 1} | v_i | && \text{desigualdade triangular} \\
&\leq \left( | c_{jj} | - | w_j | \right) + \left( | w_j | / | \alpha | \right) \left( | \alpha | - | v_j | \right) && \text{pois $A$ \'{e} diagonalmente dominante} \\
&\leq \left\| c_{jj} - w_j v_j / \alpha \right\| \\
&= \| b_{jj} \|.
\end{align*}
\end{solution}
\question Considere o sistema linear $A x = b$, $A : n \times n$ n\~{a}o singular. O objetivo \'{e} analisar as solu\c{c}\~{o}es dos sistemas: $A x = b$ e $A (x + \delta x) = b + \delta b$, (introduzimos uma pertuba\c{c}\~{a}o no vetor $b$). \'{E} desej\'{a}vel que se esta pertuba\c{c}\~{a}o for pequena ent\~{a}o a pertuta\c{c}\~{a}o na solu\c{c}\~{a}o tamb\'{e}m seja pr\'{o}xima de zero. Analisando em termos relativos aos vetores originais estamos perguntando se vale a rela\c{c}\~{a}o: $\| \delta b\| / \| b \| \approx 0 \rightarrow \| \delta x \| / \| x \| \approx 0$.
\begin{parts}
\part Demonstre que $\| \delta x \| \| x \| \leq \text{cond}(A) \| \delta b \| / \| b \|$ e analise se a rela\c{c}\~{a}o acima \'{e} verdadeira ou falsa.
\begin{solution}
\end{solution}
\end{parts}
\question[Trefethen\nocite{Trefethen:1997:numerical}, Lecture 22] Sobre o Fator de Crescimento:
\begin{quote}
\textbf{Teorema:} Supor que a fatora\c{c}\~{a}o $LU$ de $A$ foi obtida sem pivoteamento. Ent\~{a}o para todo $\epsilon_\text{maq}$, precis\~{a}o da m\'{a}quina, suficientemente pr\'{o}ximo de zero, a fatora\c{c}\~{a}o \'{e} realizada com sucesso em aritm\'{e}tica de ponto flutuante (nenhum pivô nulo \'{e} gerado) e as matrizes computadas, $\tilde{L}$ e $\tilde{U}$ satisfazem a equa\c{c}\~{a}o:
\[
\tilde{L} \tilde{U} = A + \delta A, \frac{\| \delta A \|}{\| L \| \| U \|} = \mathcal{O}(\epsilon_\text{maq}).
\]
(Informa\c{c}\~{o}es: $\epsilon_\text{maq} \approx 10^{-16}$ em precis\~{a}o dupla e $\epsilon_\text{maq} \approx 10^{-8}$ em precis\~{a}o simples.)
\end{quote}
Considerando agora a fatora\c{c}\~{a}o $LU$ de $A$ com pivoteamento parcial. Neste caso, $\| L \| = \mathcal{O}(1)$ (por qu\^{e}?). O fator de crescimento para $A$ \'{e} definido por $\rho = \max u_{ij} / \max a_{ij}$ e usando este fator e a rela\c{c}\~{a}o do teorema podemos extrair a rela\c{c}\~{a}o: $\| U \| = \mathcal{O}(\rho \| A \|)$ (deduza essa rela\c{c}\~{a}o). E, da\'{i} temos finalmente a rela\c{c}\~{a}o:
\[
\tilde{L} \tilde{U} = P A + \delta A, \frac{\| \delta A \|}{\| A \|} = \mathcal{O}(\rho \epsilon_\text{maq}).
\]
Portanto, podemos afirmar que a fatora\c{c}\~{a}o $LU$ \'{e} backward stable\footnote{Estabilidade analisada quando se compara a matriz original com a obtida ao se multiplicar $L$ por $U$.} se $\rho = \mathcal{O}(1)$.
\begin{parts}
\part Demonstre que $\rho \leq 2^{n-1}$ quando os fatores $L$ e $U$ s\~{a}o obtidos com estrat\'{e}gia de pivoteamento parcial. Conclua que este limite superior pode resultar em um fator de crescimento excessiamente grande para matrizes de grande porte.
\begin{solution}
\end{solution}
\part Efetue a fatora\c{c}\~{a}o $LU$ de $A : 5 \times 5$ com pivoteamento parcial e calcule o fator de crescimento, onde
\[
A = \begin{bmatrix}
1 & 0 & 0 & 0 & 1 \\
-1 & 1 & 0 & 0 & 1 \\
-1 & -1 & 1 & 0 & 1 \\
-1 & - 1 & -1 & 1 & 1 \\
-1 & -1 & -1 & -1 & 1
\end{bmatrix}
\]
e verifique que este limitante superior para $\rho$ pode ocorrer\footnote{O exemplo foi constru\'{i}do para provar que o pior caso pode acontecer. Em aplica\c{c}\~{o}es reais, tal fator de crescimento nunca ocorreu.}.
\begin{solution}
\end{solution}
\end{parts}
\end{questions}